Which one of the following could be the order in which the vegetables are added to the pot, listed from first to last?

ihenry20 on January 5, 2023

Game setup

If Y and P can both be out, why do you have to have P if you have J?

Reply
Create a free account to read and take part in forum discussions.

Already have an account? log in

Emil-Kunkin on January 8, 2023

Hi, I think our third rule tells us that one of Y and P must be in, and that they cannot both be out.